http://7sage.com/lsat_explanations/lsat-20-section-1-question-06/
I understand why the answer (D) is correct, but not sure why the answer choice (C) is incorrect. Is it because the answer choice makes an assumption that natural nesting sites will be ...
http://7sage.com/lsat_explanations/lsat-20-section-1-question-02/
Ok, I am pretty pissed I missed this because I felt like I got it right when I did it. I didn't even mark it for BR. As a CPA, the equivocation of a few words in the stimulus and ...
http://7sage.com/lsat_explanations/lsat-20-section-1-question-05/
After reading the answers, I think I can see how this one justifies the argument but still a little unsure about how the answer was found.
http://7sage.com/lsat_explanations/lsat-20-section-1-question-20/
I got this question correct since C was obviously not relevant to the argument, but during BR, I can't seem to eliminate B or E. Why are these necessary assumptions? Here is my ...
Hey 7Sage, I'm having the toughest time understanding why Answer B is the correct answer for this question. In reviewing my original answer choice, D, I can understand why I was incorrect in choosing it, but am caught up between answer choices B and C. Why ...
I have a question on answer choice C.
I understand the flaw of the survey: how it fails to distinguish the residents who dropped out in its own schools and those who dropped out of schools from somewhere else. But, after ...
(P.S., I know this is a long and dense post, but there's an opportunity at the bottom for anyone reading this to get paid, so hopefully that's an incentive to read this :P)
Hi guys! I wanted to get some feedback from you smart people on the ...
Can someone help me to identify opponent's argument, since I don't see clearly where is a conclusion and where are premises. All I see is a set of premises with unstated(assumed) conclusion. Thank you!! #help
Can someone please explain how did we get C as correct AC. We don't have to have same totals in order to get same percentages. Why does this question requires us to make this assumption? Thank you!
My issue is in the the stimulus finding the conclusion. Can Someone please help?
**Admin note: edited title; please use the format of "PT#.S#.Q# - [brief description]"**
https://7sage.com/lsat_explanations/lsat-41-section-1-question- ...